1answer.
Ask question
Login Signup
Ask question
All categories
  • English
  • Mathematics
  • Social Studies
  • Business
  • History
  • Health
  • Geography
  • Biology
  • Physics
  • Chemistry
  • Computers and Technology
  • Arts
  • World Languages
  • Spanish
  • French
  • German
  • Advanced Placement (AP)
  • SAT
  • Medicine
  • Law
  • Engineering
DanielleElmas [232]
2 years ago
9

Solve for Y Answers choices Y=120 Y=60 Y=20 Y=10

Mathematics
1 answer:
fomenos2 years ago
3 0

Answer:I’m going with 60 but I might be wrong cuz I did this in my head and I am not good with mental math so

Step-by-step explanation:

You might be interested in
There are 100 sheets of paper in the printer, and the number of sheets of paper, p, left after ,t, minutes of printing is given
Mice21 [21]
I think so 12 to 13 minutes will it take to use all sheets..
8 0
3 years ago
The formula for the volume of a sphere is a power function:
soldier1979 [14.2K]

Answer:

r≥0

V(r) ≥0

Step-by-step explanation:

V(r) = 4/3 * Pi* r^3

The domain is the input values for r

The radius cannot be less than 0  but can be any number bigger than 0

What is the domain of the function in this situation?

r≥0

The range is the output values for the function or the volume

The volume must be greater than or equal to zero

V(r) ≥0

7 0
3 years ago
Read 2 more answers
Plz hurry help!!!!:) What is the length of AD? Round your answer to the nearest hundredth.
jolli1 [7]

Answer:

AD=5.82 units

Step-by-step explanation:

tan 31=3/BD

0.6009=3/BD

BD=5

AD^2=3^2 + 5^2

AD^2=9+25=34

AD=5.82

8 0
3 years ago
Read 2 more answers
What is the greatest value of 2,463.9051
RoseWind [281]

Hey There @Bre18016,

The answer is \boxed{2}

The greatest value of 2,463.9051 would be the thousands place (2) simply as it is the biggest number out of the other places.

For instance, if we had the number 300, 3 would be the greatest value.

Or let's say we had 10,000 the 1 would be the greatest value.

Furthermore, you could look at the first digit in the entire number to deter mine the greatest value.

6 0
3 years ago
Did I do this right?
Rasek [7]

Answer:

Yea u did......

Step-by-step explanation:

you did right because all the steps are right

5 0
3 years ago
Read 2 more answers
Other questions:
  • Which of the following have two congruent parallel bases?
    5·1 answer
  • Graham uses a 10 by 10 grid to model a decreasing pattern. He cuts off a section representing a percent of the current area so t
    9·2 answers
  • There are 20 wild pigs on an island and the number of pigs doubled each year for the past 5 years. The independent variable is
    9·2 answers
  • Sherwin has 3 cups of grapes he likes to pack 3/4 cup of fruit in his lunch each day how many days can Sherwin pack grapes in hi
    12·2 answers
  • In which quadrant does -8,-10 lie?
    9·1 answer
  • What is 10000 x 10 im confused number is too big
    8·2 answers
  • Suppose the number of messages that an inbox receives may be modeled by a Poisson distribution. If the average number of message
    10·1 answer
  • Last question for the night.
    9·2 answers
  • True or False?
    11·2 answers
  • How can i ask questions ?​
    6·1 answer
Add answer
Login
Not registered? Fast signup
Signup
Login Signup
Ask question!